0 Daumen
1,1k Aufrufe

ich weiß, dass beide Folgen gegen e gehen, aber ich weiß nicht wie man diese Behauptung zeigen könnte.


$$(1+\frac { 1 }{ n } )^{ n }\quad <\quad { (1+\frac { 1 }{ n }  })^{ n+1 }$$

Avatar von

( 1 + 1/n ) < ( 1 + 1/n )^{n+1}
( 1 + 1/n ) < ( 1 + 1/n )^{n} * ( 1 + 1/n )^{1}
1 < ( 1 + 1/n )^{1}
1 < 1 + 1/n
0 < 1/n

Danke für deine Bemühungen, leider hast du mit 1+1/n und nicht mit (1+1/n)^n gerechnet. Die Lösung von Mathef ist aber Korrekt.

Das hast du recht. ich korrigiere

( 1 + 1/n )^n < ( 1 + 1/n )^{n+1}
( 1 + 1/n )^n < ( 1 + 1/n )^{n} * ( 1 + 1/n )^{1}
1 < ( 1 + 1/n )^{1}
1 < 1 + 1/n
0 < 1/n

1 Antwort

0 Daumen
 
Beste Antwort
Teile doch einfach die Ungleichung durch   (1 + 1/n)^n , das kannst du, weil das positiv ist.
Dann hast du   1 < 1+ 1/n und das stimmt für jedes n.
Avatar von 288 k 🚀

Ein anderes Problem?

Stell deine Frage

Willkommen bei der Mathelounge! Stell deine Frage einfach und kostenlos

x
Made by a lovely community